Question
Why is c not the answer. Why is the correct answer d?
Eunbeezy on November 19, 2014
  • October 2004 LSAT
  • SEC4
  • Q15
1
Reply
Help
Please explain why the answer goes to (E), not to (a) or (c). Anyhow, this is an assumption quest...
Batman on November 18, 2014
  • June 1998 LSAT
  • SEC2
  • Q24
1
Reply
Recommendations for attacking this problem
Do you have any recommendations on how to most effectively attack this problem. I noticed with th...
MJA7 on November 18, 2014
  • December 2007 LSAT
  • SEC2
  • Q22
1
Reply
Revised question
Is the first sentence of the last paragraph(48~50) clue for the answer (A)? If it is, how can tes...
Batman on November 18, 2014
  • June 1998 LSAT
  • SEC1
  • Q10
1
Reply
Help
Please explain why (A) is the answer. Thanks.
Batman on November 17, 2014
  • June 1998 LSAT
  • SEC1
  • Q10
1
Reply
Please Help
Please Explain
KDA86 on November 17, 2014
  • December 2004 LSAT
  • SEC2
  • Q9
1
Reply
Explanation help
Please help explain why A is correct. I believe the discrepancy with the passage is that the conc...
MJA7 on November 17, 2014
  • October 2008 LSAT
  • SEC3
  • Q22
2
Replies
Help
Please explain why (E) could not be the answer. Thanks,
Batman on November 13, 2014
  • December 1997 LSAT
  • SEC3
  • Q12
1
Reply
Explanation help
Please explain why the answer is C. I narrowed it down to C and A and reasoned that because A spe...
MJA7 on November 13, 2014
  • September 2007 LSAT
  • SEC1
  • Q21
1
Reply
Help
Please explain why A is a better answer than D. Thanks!!!
MJA7 on November 12, 2014
  • October 2008 LSAT
  • SEC3
  • Q17
1
Reply
Help
Please explain why (a) is the answer.
Batman on November 11, 2014
  • December 1997 LSAT
  • SEC2
  • Q21
1
Reply
Question answer help
Can you help explain why E is correct and not C
MJA7 on November 10, 2014
  • September 2007 LSAT
  • SEC1
  • Q19
1
Reply
Could you,show me how to diagram this game?
Thanks! You guys are great.
Jprince on November 7, 2014
  • October 2004 LSAT
  • SEC3
  • Q20
1
Reply
Assistance needed
Hi, I could use some help with this specific question. I am confused because when T speaks secon...
Titan on November 6, 2014
  • October 1997 LSAT
  • SEC1
  • Q23
2
Replies
Set-up and problem solving
I would like to see the set-up for all the logic games on this practice test (PT 60). Is this mat...
MJA7 on November 6, 2014
  • June 2010 LSAT
  • SEC2
  • Q1
3
Replies
Question help
I am having difficulty with understanding why the answer is E. Can you help explain why the answe...
MJA7 on November 4, 2014
  • September 2007 LSAT
  • SEC1
  • Q17
2
Replies
Help?
How does D weaken the argument ?
sairaj87 on November 3, 2014
  • December 2013 LSAT
  • SEC3
  • Q21
1
Reply
Help
I chose A as the answer. Not sure why it's B!
sairaj87 on November 3, 2014
  • December 2013 LSAT
  • SEC3
  • Q22
1
Reply
Help
Explanation??
sairaj87 on October 29, 2014
  • December 2013 LSAT
  • SEC3
  • Q24
1
Reply
Help
I really have trouble with strengthening questions. They almost always are so close! Missed this ...
sairaj87 on October 29, 2014
  • December 2013 LSAT
  • SEC3
  • Q25
1
Reply